Магнитные монополи Дирака и дробное квантование электрического заряда кварков

Применяя правило квантования Дирака для электрического и магнитного заряда, я предполагаю, что рассматриваются единичные электрические заряды, такие как электроны. Как правило квантования Дирака применяется к дробным электрическим зарядам кварков?

Я отвечаю на этот вопрос на physics.stackexchange.com/q/268709 .

Ответы (2)

Правило квантования Дирака происходит от интегрирования углового момента наложенного электромагнитного поля заряда и монополя. Этот угловой момент оказывается конечным и не зависит от расстояния час между зарядом и монополем. Затем аргумент состоит в том, что если возможно выделить один фундаментальный заряд е и единственный фундаментальный монополь г в некоторой области пространства, то полный угловой момент в этой области должен быть кратен . Здесь «изоляция» означает, что расстояние до любой другой частицы равно час .

Но обратите внимание, что изоляция частиц имеет решающее значение. Если вы поместите монополь рядом с атомом водорода, полный угловой момент электромагнитного поля исчезнет, ​​потому что плотность углового момента имеет вид Е × Б , который является билинейным по полям.

Поскольку кварки ограничены, этот аргумент никогда не может быть применен к кварку.

Это хорошая точка. Тем не менее, этот аргумент применим к КТП, где количество разновидностей достаточно велико, чтобы не происходило ограничения.
Что, если поднять температуру выше температуры Хагедорна ? Тогда кварки деконформируются, и вы сможете перемещать один кварк по струне Дирака.
Ваш аргумент неприменим в случае, когда вы перемещаете кварк вокруг монополя по окружности меньше радиуса удержания или выше радиуса Хагедорна. Полный ответ см . на physics.stackexchange.com/questions/268709/… .

Логика та же: если д представляет собой электрический заряд и г является магнитным зарядом, необходимо иметь г д е 2 π Z (в единицах ленивого теоретика). Итак, если есть наибольший магнитный заряд, то обязательно есть и наименьший электрический заряд. Вам не нужно предполагать, что вы имеете дело с удельной стоимостью, чтобы привести этот аргумент.